PA - Geriatric and a few odd ball board prep

Réussis tes devoirs et examens dès maintenant avec Quizwiz!

The glomerular filtration rate (GFR) decreases by approximately ____ ml/min/year after age 40. A. 1 B. 2 C. 3 D. 4

A. 1 After age 40, GFR lowers approximately 1 mL/min/year after age 40.

Parkinson's disease is associated with A. A decrease in dopamine levels in the basal ganglia B. A decrease in dopamine levels in the cerebellum C. An increase in dopamine levels in the basal ganglia D. An increase in dopamine levels in the cerebellum

A. A decrease in dopamine levels in the basal ganglia Parkinson's disease is associated with a loss of dopaminergic fibers and subsequently, decreased dopamine levels in the basal ganglia. The result is unopposed action of acetylcholine and a decrease in the inhibition of the extrapyramidal motor system.

What blood pressure is designated as a hypertensive emergency in the parturient? A. A diastolic greater than 109 mmHg B. A diastolic greater than 129 mmHg C. A systolic pressure greater than 149 mmHg D. A systolic pressure greater than 179 mmHg

A. A diastolic greater than 109 mmHg Encephalopathy usually does not present until the diastolic blood pressure reaches 150 mmHg. The exception is parturients who may exhibit signs of encephalopathy with diastolic pressures at 100 mmHg. Even if no overt signs of end-organ damage are noted, a diastolic pressure higher than 109 mmHg in a parturient is designated as a hypertensive emergency and immediate treatment is warranted.

Which of the following characteristics do geriatric patients share with neonates? A. A higher body surface area to body mass ratio B. A higher relative concentration of body water C. Decreased alpha-1 acid glycoprotein levels D. Increased skeletal muscle mass

A. A higher body surface area to body mass ratio Both classes of patient have a high body surface area to body mass ratio. Although neonates have a higher concentration of body water, geriatric patients have a lower proportion of body water. Although neonates can have a lower alpha-1 acid glycoprotein level than adults, this value is either normal or slightly elevated in geriatric patients.

Which of the following would best exemplify a diagnosis of hypertensive emergency? A. A parturient with a blood pressure of 179/110 an no other symptoms B. A patient with a blood pressure of 185/118 and epistaxis C. A patient with a blood pressure of 192/105 and headache D. A patient with a blood pressure of 179/100 and anxiety

A. A parturient with a blood pressure of 179/110 an no other symptoms Hypertensive emergency is defined as hypertension with evidence of end-organ damage such as myocardial ischemia, dissecting aortic aneurysm, renal insufficiency, pulmonary edema, encephalopathy, eclampsia, or intracerebral hemorrhage. Hypertension without signs of end-organ damage is termed 'hypertensive urgency'. These patients often present with hypertension and symptoms such as headache, epistaxis, or anxiety. The exception to the rule is parturients. A parturient with a diastolic blood pressure greater than 109 mmHg is defined as being in a state of hypertensive emergency even if no other symptoms are present.

What test is used to assess the effects of heparin and protamine during cardipulmonary bypass surgery? A. Activated clotting time B. Prothrombin time C. Platelet count D. Factor VIII levels

A. Activated clotting time During cardiac bypass, heparinization is monitored by measuring the activated clotting time.

What coagulation factors are present in fresh frozen plasma? A. All coagulation factors B. All except factor X C. All except factor VII D. All except von Willebrand factor

A. All coagulation factors Complement, fibrinogen, albumin, globulins, and all coagulation factors are present in fresh frozen plasma. It may be administered for reversal of the effects of warfarin, vitamin K deficiency, bleeding due to dilutional coagulopathy, or treatment of microvascular bleeding due to prolonged prothrombin time or partial thromboplastin time.

Which of the following respiratory parameters is decreased in elderly patients? A. Alveolar surface area B. Residual volume C. Closing capacity D. Both A and B

A. Alveolar surface area In the elderly patient, closing capacity increases, and by age 65 it exceeds tidal volume in the sitting position resulting in airway collapse. The increase in airway collapse increases residual volume as well. The alveolar surface area decreases with age resulting in decreased efficiency of gas exchange.

Which drug could potentiate the effects of atropine and predispose the patient to central anticholinergic syndrome? A. Amitryptiline B. Morphine C. Pancuronium D. Ketamine

A. Amitryptiline Tricyclic antidepressants (like amitryptiline), antipsychotics, and antihistamines have antimuscarinic characteristics which can potentiate anticholinergics and predispose the patient to central anticholinergic syndrome.

The normal response of the juxtaglomerular apparatus to chronic vasoconstriction is A. An increase in the release of renin B. A decrease in the release of renin C. An increase in the release of aldosterone D. A decrease in the release of aldosterone

A. An increase in the release of renin In the face of chronic vasoconstriction, the juxtaglomerular apparatus releases renin. The end result of renin release is the conversion of angiotensin I to angiotensin II. Angiotensin II is the primary stimulus for the release of aldosterone by the adrenal cortex. Aldosterone results in increased sodium and water retention.

Which of the following drugs would be least appropriate for use in a patient with advanced Alzheimer's disease? A. Atropine B. Glycopyrrolate C. Ephedrine D. Phenylephrine

A. Atropine Alzheimer's is associated with a progressive and selective decrease in cholinergic neurons in the brain. For this reason, centrally acting anticholinergics such as atropine and scopolamine are not recommended for use.

Which of the following would be appropriate in the anesthetic management of a patient with hypertrophic cardiomyopathy? A. Avoid an increase in myocardial contractility B. Maintain a slightly increased heart rate C. Restrict IV fluids to decrease the preload D. Administer nitrates to decrease the afterload

A. Avoid an increase in myocardial contractility Hypertrophic cardiomyopathy involves enlargement of the interventricular septum which results in left ventricular outflow obstruction. This obstruction is worsened by increased heart rate or increased myocardial contractility as well as decreases in preload or afterload. Anesthesia is usually maintained by controlled myocardial depression using volatile anesthetics.

Which of the following increases as a result of the aging process? A. Catecholamine levels B. Parasympathetic activity C. Response to beta receptor stimulation D. The efficacy of baroreflex control of blood pressure

A. Catecholamine levels As a result of aging, the response to beta-receptor stimulation and parasympathetic activity decrease. Serum catecholamine levels increase as a result of aging. Efficacy of blood pressure control by the baroreflex decreases with age.

Which anatomic structure is responsible for formation of aqueous humor? A. Ciliary body B. Schlemm's canal C. Corneal epithelium D. Ciliary ganglion

A. Ciliary body Two-thirds of the aqueous humor in the eye is formed by the ciliary body in the posterior chamber of the eye. The remaining third is formed by passive filtration from vessels through the anterior surface of the iris.

All of the following physiologic parameters decrease in the elderly except A. Closing volume B. Renal mass C. Lean body mass D. Body water

A. Closing volume Compared with younger adults, the geriatric population have a reduced lean body mass, decreased total body water, decreased serum albumin, decreased kidney mass, and decreased hepatic blood flow. Body fat and closing volume, however, increase with age.

What is the most common cause of heart failure? A. Coronary artery disease B. Valvular disease C. Arrhythmias D. Pericardial disease

A. Coronary artery disease Impaired contractility due to ischemic heart disease or cardiomyopathy are the most common culprits, but valvular disease, hypertension, pericardial disease, and pulmonary hypertension are also causes of heart failure.

Which of the following changes would you expect to occur normally in elderly patients? A. Decreased serum albumin B. Increased MAC C. Decreased serum alpha-1 acid glycoprotein D. Increased total body water

A. Decreased serum albumin In elderly patients, serum albumin levels decrease resulting in increased free serum levels of acidic drugs. MAC decreases approximately 6% per decade after 40 years of age, and total body water also decreases.

Factors that decrease the amount of prolapse in patients with mitral valve prolapse include (select four) A. Drug-induced myocardial depression B. Pulmonary hypertension C. Increased preload D. Hypertension E. Decreased systemic vascular resistance F. Tachycardia G. Atrial fibrillation H. Vasoconstriction

A. Drug-induced myocardial depression C. Increased preload D. Hypertension H. Vasoconstriction Any factor that maintains a larger ventricular volume will decrease the degree of prolapse. Hypertension, vasoconstriction, drug-induced myocardial depression, and increased preload will decrease the degree of prolapse.

During your patient interview in the preoperative area, you noticed that the patient's chart indicates a diagnosis of right atrial hypertrophy. In lead II of the ECG, which characteristic is consistent with right atrial hypertrophy? A. Enlarged P wave B. Depressed P wave C. Inverted P wave D. Widened QRS complex

A. Enlarged P wave The presence of an enlarged P wave in lead II, also known as P pulmonale, is consistent with right atrial hypertrophy.

A patient has been diagnosed with an ischemic stroke. Which agent should be avoided in the care of this patient? A. Glucose B. Labetolol C. Tissue plasminogen activator D. Heparin

A. Glucose Hyperglycemia is associated with poor outcomes in patients with ischemic stroke. Intravenous administration of glucose should be avoided in stroke patients. Labetolol is often used to control blood pressure in these patients. Tissue plasminogen activator and heparin may be used in the treatment of an ischemic stroke.

Which of the following statements regarding heparin-induced thrombocytopenia (HIT) syndrome are true? A. HIT syndrome occurs in about 5% of patients receiving heparin B. HIT syndrome usually develops within 6 hours of initiating heparin therapy C. HIT syndrome is mediated by IgE antibodies D. Both A and C are true

A. HIT syndrome occurs in about 5% of patients receiving heparin Heparin-induced thrombocytopenia syndrome is a potentially life-threatening disorder that occurs in about 5% of patients receiving heparin. The patient develops severe thrombocytopenia, becomes resistant to the effects of heparin, and may develop thombosis despite the low platelet count. It usually develops after 6-10 days of heparin therapy. It is due to the development of platelet-associated immunoglobulin (IgG) antibodies.

Prothrombin complex concentrates contain a concentration of factors A. II, VII, IX, and X B. VIII and IX C. II and V D. II and IV

A. II, VII, IX, and X Prothrombin complex concentrates contain a concentration of factors II, VII, IX, and X. It can be used in the treatment of warfarin reversal and hemophilia B, but it is used for many off-label purposes as well. Compared to fresh frozen plasma, PCCs resolve the INR more quickly and do not require crossmatching.

Which of the following hemodynamic effects would you most expect to see in a patient placed on positive pressure ventilation? A. Increased right ventricular afterload B. Increased right ventricular stroke volume during inspiration C. Increased LVEDV during expiration D. Increased central venous pressure

A. Increased right ventricular afterload Positive pressure ventilation reduces venous return, increases right ventricular afterload, decreases right ventricular stroke volume during inspiration, and decreases LVEDV during expiration

Select the two most prominent effects of aging on the pulmonary system. A. Increased stiffness of the chest wall B. Lowered work of breathing C. Decreased stiffness of the parenchyma D. Reduction in upper airway diameter

A. Increased stiffness of the chest wall C. Decreased stiffness of the parenchyma The two most prominent effects of increased age on the pulmonary system are a decrease in the stiffness of the lung parenchyma and an increase in the stiffness of the chest wall.

Which of the following represents a primary aim of the treatment of Parkinson's disease? A. Increasing the amount of dopamine in the basal ganglia B. Increasing the neuronal effects of acetylcholine C. Increasing the amount of serotonin in the neuronal synapse D. Preventing systemic hypertension

A. Increasing the amount of dopamine in the basal ganglia Treatment of Parkinson's disease is aimed at increasing the amount of dopamine in the basal ganglia or suppressing the neuronal effects of acetylcholine.

How does an increase in the diastolic blood pressure affect the severity of aortic regurgitation? (select two) A. It results in an increase in the backward pressure gradient across the valve B. It results in a decrease in the backward pressure gradient across the valve C. It decreases the left ventricular regurgitant volume D. It increases the left ventricular regurgitant volume

A. It results in an increase in the backward pressure gradient across the valve D. It increases the left ventricular regurgitant volume An increase in the diastolic blood pressure increases the backward pressure gradient which results in an increase in the proportion of stroke volume that regurgitates back into the left ventricle.

Which of the following is not associated with septic shock? A. Low cardiac output B. Low systemic vascular resistance C. Tissue hypoperfusion D. Disseminated intravascular coagulation

A. Low cardiac output Septic shock is typically characterized by a high cardiac output, low systemic vascular resistance, and tissue hypoperfusion. Endotoxins from the bacteria can affect the coagulation pathway, resulting in DIC. DIC is one of the reasons that the mortality rate for septic shock is as high as 85 percent.

Which ventilator setting guidelines would be the most appropriate for the patient with sepsis? A. Low tidal volume, low inspiratory pressure, and +5 of PEEP B. Low tidal volume, low inspiratory pressure, avoid PEEP C. High tidal volume, low inspiratory pressure, and +5 of PEEP D. High tidal volume, moderate inspiratory pressure, avoid PEEP

A. Low tidal volume, low inspiratory pressure, and +5 of PEEP The current recommended protocol is a low tidal volume, low inspiratory pressure, and the application of PEEP.

Which disorder exhibits a higher incidence of mitral valve prolapse? A. Marfan syndrome B. COPD C. Myasthenia gravis D. Stevens-Johnson syndrome

A. Marfan syndrome MVP has a higher occurrence in patients with Marfan syndrome, systemic lupus erythematosus, rheumatic carditis, thyrotoxicosis, and myocarditis.

Which narcotic has been reported to produce hypertensive crisis, convulsions, and coma in patients taking MAO inhibitors? A. Meperidine B. Fentanyl C. Alfentanil D. Hydromorphone

A. Meperidine Meperidine has been identified to produce hypertensive crisis, seizures, and coma in patients taking MAO inhibitors.

What changes in heart sounds are associated with mitral valve prolapse? (select two) A. Midsystolic click B. Late diastolic murmur C. Opening snap D. Late systolic murmur

A. Midsystolic click D. Late systolic murmur Mitral valve prolapse is associated with a midsystolic click and a late systolic murmur. An opening snap is associated with mitral stenosis.

Which alteration would be most likely to produce congestive heart failure or hypotension in the patient with aortic stenosis? A. New onset of atrial fibrillation B. Decrease in heartrate from 70 bpm to 50 bpm C. PVCs at a rate of 6-8 per minute D. New onset right bundle branch block

A. New onset of atrial fibrillation The decreased pressure gradient that exists between the left atrium and left ventricle in aortic stenosis limits left ventricular filling dramatically. Because left ventricular filling is so dependent upon atrial contraction, loss of atrial systole can result in congestive heart failure or hypotension.

Which of the following pharmacologic treatments may result in increased intracranial pressure when treating hypertensive emergencies associated with encephalopathy? A. Nitroprusside B. Esmolol C. Labetalol D. Hydralazine

A. Nitroprusside Nitroprusside may increase intracranial pressure and therefore must be used cautiously in the treatment of hypertensive crises associated with encephalopathy.

Which of the following ECG changes are associated with cor pulmonale? A. Peak P waves in leads II, III, and aVF B. Left axis deviation C. Left bundle branch block D. Inverted T wave in leads I and aVL

A. Peak P waves in leads II, III, and aVF The ECG changes associated with cor pulmonale are due to the increase in mass of the right side of the heart. Right atrial hypertrophy is manifested as peak P waves in leads II, III, and aVF. Right axis deviation and right bundle branch block are often present as a result of right ventricular hypertrophy.

Which of the following electrocardiographic changes are consistent with cor pulmonale? A. Peaked P waves in leads II, III, and aVF B. Ventricular bigeminy C. Deep S waves in lead V1 and tall R waves in lead V5 D. Biphasic P waves in lead V1

A. Peaked P waves in leads II, III, and aVF Electrocardiographic signs consistent with cor pulmonale include peaked P waves in leads II, III, and aVF which are consistent with right atrial hypertrophy and right axis deviation and right bundle branch block which are consistent with right ventricular hypertrophy.

What is the most common cause of bleeding after surgery? A. Platelet dysfunction B. Low fibrinogen levels C. Factor V deficiency D. Factor XIII deficiency

A. Platelet dysfunction Platelet dysfunction is the most common cause of bleeding following surgery.

What preoperative criteria is the strongest predisposing factor to postoperative delirium? A. Preexisting dementia B. Renal impairment C. Hepatic impairment D. The use of benzodiazepines for preoperative sedation

A. Preexisting dementia Preoperative dementia is the strongest predictor for postoperative delirium in the elderly. Other predisposing factors include dehydration, alcohol use, psychoactive drug use, and impaired vision.

Which issues would you expect to see in a geriatric patient? (select two) A. Problems eliminating excess sodium B. Problems retaining sodium C. Decreased alpha-1 acid glycoprotein levels D. Increased serum albumin levels

A. Problems eliminating excess sodium B. Problems retaining sodium In elderly patients, the kidneys are not as effective at eliminating excess sodium, nor are they as effective at retaining sodium. This is due primarily to a reduction in the secretion of aldosterone. Serum albumin decreases slightly with age, but alpha-1 acid glycoprotein levels are usually unchanged or even experience a slight increase.

Which antiarrhythmic drug is least appropriate for use in patients with congestive heart failure? A. Procainamide B. Amiodarone C. Lidocaine D. Esmolol

A. Procainamide The class IA (quinidine, procainamide) and IC antiarrhythmics (flecainide, propafenone) are not recommended for use in patients with congestive heart failure because of their negative inotropic effects and tendency to be proarrhythmic in these patients. Amiodarone is considered safe in these patients.

Multiple sclerosis is associated with which of the following? A. Prolonged latency of evoked potentials B. Excessive accumulation of myelin on the axon C. Neuropathy due to demyelination of the peripheral nerves D. Rapid progression in patients diagnosed after 35 years of age

A. Prolonged latency of evoked potentials Multiple sclerosis is associated with prolonged latency of evoked potentials due to slowed nerve conduction resulting from demyelination of central nerves, including the brainstem and cerebellum. It is characterized by a slow progression when the onset occurs after 35 years of age. It does not result in excessive accumulation of myelin, nor does it affect the peripheral nerves.

What cross allergy is of concern with a DM patient on NPH insulin presenting for cardiac surgery? A. Protamine B. Cefazolin C. Morphine D. Midazolam

A. Protamine Protamine exhibits an increased risk of allergic reaction in patients taking NPH insulin.

Which of the following is not true of patients with scleroderma? A. Raynaud's phenomenon is rarely associated with scleroderma B. Gastrointestinal hypermotility is rarely associated with scleroderma C. Tracheal intubation is likely to be difficult D. Scleroderma may result in chronic hypoxemia

A. Raynaud's phenomenon is rarely associated with scleroderma This question simulates the mixture of combination negative and double-negative scenarios that may appear on the nurse anesthesia exam and warrant close scrutiny to avoid confusion, especially in the high-stress period that accompanies an exam of this magnitude. In this example, all of the above statements are true except option A. Over 95% of patients with scleroderma experience Raynaud's phenomenon

Which of the following alterations would you expect to see in a patient treated with acetylcholinesterase inhibitors for Alzheimer's disease? A. Resistance to nondepolarizing muscle relaxants B. Shorter duration of action with succinylcholine C. Greater tendency to exhibit baseline tachycardia D. Decreased incidence of nausea

A. Resistance to nondepolarizing muscle relaxants Acetylcholinesterase inhibitors are the most common treatment for Alzheimer's and may result in nausea, vomiting, bradycardia, syncope, and fatigue. They will exhibit a resistance to nondepolarizing muscle relaxants due to the use of acetylcholinesterase inhibiting drugs as part of their medical management. They will also exhibit a longer duration of action with succinylcholine.

Which of the following ECG changes is an indication of subendocardial ischemia? A. ST depression B. ST elevation C. Inverted T waves D. Widened QRS wave

A. ST depression Depression of the ST segment is typically associated with subendocardial ischemia. ST segment elevation is typically associated with transmural ischemia.

Which of the following 12-lead electrocardiographic changes would be most consistent with subendocardial injury? A. ST depression in lead II B. ST elevation in lead II C. Shortened PR interval in lead I D. ST elevation in lead I

A. ST depression in lead II ST depression that coincides in time with anginal symptoms is consistent with subendocardial injury.

What ECG changes are associated with a subarachnoid hemorrhage? (select two) A. ST segment depression B. Widened QRS C. Heart block D. T wave inversion

A. ST segment depression D. T wave inversion ST segment depression and T wave inversion are often noted within 48 hours of a subarachnoid hemorrhage and are associated with the increase in catecholamine release.

What is one reason many geriatric patients are more susceptible to hypothermia? A. They are more prone to hypothyroidism B. The have a lower body surface area to body mass ratio C. They have a higher basal metabolic rate D. They have an increase in lean muscle mass

A. They are more prone to hypothyroidism Geriatric patients are more susceptible to hypothermia because they have a lower basal metabolic rate, a higher incidence of hypothyroidism, a high proportion of body surface area to body mass, and their ability to produce peripheral vasoconstriction to reduce heat loss is not as effective.

Which of the following statements regarding the use of muscle relaxants in patients with Huntington's disease is true? A. They are sensitive to both succinylcholine and nondepolarizing muscle relaxants B. Succinylcholine is contraindicated, but the response to nondepolarizing muscle relaxants is normal C. The response to succinylcholine is normal, but they are sensitive to nondepolarizing muscle relaxants D. The response to both depolarizing and nondepolarizing muscle relaxants is normal

A. They are sensitive to both succinylcholine and nondepolarizing muscle relaxants Patients with Huntington's Chorea have extreme sensitivity to nondepolarizing muscle relaxants, therefore they should be titrated very carefully. Also, because of a decrease in plasma cholinesterase, they exhibit an increased sensitivity to succinylcholine.

Which of the following statements accurately explains why geriatric patients have higher insulin requirements perioperatively? A. They exhibit a lower insulin response to glucose loads B. They exhibit decreased insulin resistance in skeletal muscle C. They have decreased hepatic mass D. They are most likely already taking insulin

A. They exhibit a lower insulin response to glucose loads Elderly patients tend to have higher insulin requirements perioperatively, even when a diagnosis of diabetes is not present. They exhibit a lower insulin response to glucose loads as well as increased insulin resistance in skeletal muscle.

Many central nervous system neurotransmitters decline with age. Which one is most closely associated with the development of Alzheimer's disease? A. acetylcholine B. GABA C. dopamine D. serotonin

A. acetylcholine CNS concentrations of dopamine, serotonin, GABA, and acetylcholine all decrease with age. Acetylcholine is the most significant as it is linked with the development of Alzheimer's disease.

Myocardial infarction, papillary muscle dysfunction, chordae tendinae rupture, trauma to the chest, and infectious endocarditis are the most common causes of A. acute mitral regurgitation B. pulmonary hypertension C. chronic aortic stenosis D. Ebstein's anomaly

A. acute mitral regurgitation Acute mitral regurgitation can occur as a result of myocardial infarction, papillary muscle dysfunction, chordae tendinae rupture, trauma to the chest, or infectious endocarditis.

The hallmark pathologic findings in Alzheimer's such as neurofibrillatory tangles and neuritic plaques are due to A. deposition of amyloid plaques B. alpha-1 acid glycoprotein deposition C. alpha-1 antitrypsin deficiency D. glucose-6 phosphate deficiency

A. deposition of amyloid plaques The hallmark pathologic findings in Alzheimer's disease are due to beta-amyloid deposition. Alpha-1 acid glycoprotein is one of the principal proteins responsible for drug binding. Patients with homozygous alpha-1 antitrypsin deficiency develop a slowly progressive liver disease that typically results in cirrhosis. Glucose-6 phosphate deficiency is an autosomal recessive glucose metabolism defect associated with liver enlargement, and possible hypoglycemia, acidosis, and convulsions.

Phenytoin is especially useful in the treatment of arrhythmias due to A. digitalis toxicity B. myocardial infarction C. re-entrant tachycardia D. aortic valve dysfunction

A. digitalis toxicity Phenytoin is especially useful in the treatment of arrhythmias due to digitalis toxicity. It is not quite as effective as quinidine or lidocaine in the treatment of ventricular arrhythmias due to other causes and is not very effective at treating atrial arrhythmias.

Patients taking levodopa for the treatment of Parkinson's disease may exhibit (select two) A. dyskinesias B. orthostatic hypotension C. decreased risk for nausea and vomiting D. decreased cardiac contractility

A. dyskinesias B. orthostatic hypotension Levodopa is a precursor to dopamine. It is combined with a decarboxylase inhibitor to prevent the peripheral conversion of levodopa to dopamine and increase levels in the central nervous system. Side effects include dyskinesias (in over 80% of patients after one year of treatment), hallucinations, paranoia, and mania. Increases in cardiac contractility and heart rate occur. Orthostatic hypotension is also common in these individuals and levodopa therapy may result in nausea and vomiting as a result of stimulation of the chemoreceptor trigger zone

The most common form of hypertension is A. essential hypertension B. secondary hypertension C. hypertensive emergency D. prehypertension

A. essential hypertension Secondary hypertension is an increase in blood pressure due to a cause that can be identified and cured such as pheochromocytoma, renal artery stenosis, coarctation of the aorta, Conn's syndrome, or Cushing's disease. Essential hypertension is an increased blood pressure for which there is no identifiable cause and is diagnosed based on the exclusion of causes such as those listed above. Approximately 95% of patients with hypertension are diagnosed with essential hypertension. Prehypertension is defined as a systolic blood pressure between 120-139 or a diastolic blood pressure between 80-89. A diastolic pressure higher than 109 mmHg with signs of end-organ damage is designated as a hypertensive emergency. A diastolic pressure higher than 109 mmHg in the parturient is designated as hypertensive emergency even if no signs of end-organ damage are present.

Which antihypertensive agent has the shortest plasma half-life? A. fenoldopam B. hydralazine C. metoprolol D. nicardipine

A. fenoldopam Fenoldopam has a half-life of about 5 minutes and requires administration as an infusion. Metoprolol has a have life of 3-7 hours. Hydralazine has a half-life of 2-8 hours. Nicardipine has a half-life of 2-4 hours

The most common cardiac side effects of tricyclic antidepressants are (select two) A. increased heart rate B. heart block C. orthostatic hypotension D. atrial fibrillation

A. increased heart rate C. orthostatic hypotension Tricyclic antidepressants have a narrow therapeutic profile and can produce anticholinergic side effects, sedation, and cardiac effects. The most common cardiac side effects of tricyclic antidepressants are increased heart rate and orthostatic hypotension.

Class I antidysrhythmics work primarily by A. inhibiting fast sodium ion channels B. inhibiting potassium ion channels C. inhibiting slow calcium channels D. decreasing the rate of depolarization

A. inhibiting fast sodium ion channels Antidysrhythmics are divided into four major classes. Class I drugs such as quinidine, lidocaine, and flecainide inhibit fast sodium channels. Class II drugs such as esmolol and propranolol decrease the rate of depolarization. Class III drugs such as amiodarone inhibit potassium ion channels. Class IV drugs include the calcium channel blockers which work by inhibiting slow calcium channels.

Compared to pure beta-adrenergic blockers, labetolol produces (select two) A. less bradycardia B. an increased incidence of bronchospasm C. an increased risk for hypoglycemia D. an increased incidence of orthostatic hypotension

A. less bradycardia D. an increased incidence of orthostatic hypotension Because of its alpha-adrenergic blocking capability, labetolol produces less bradycardia than pure beta-adrenergic blockers, but has an increased incidence of orthostatic hypotension.

The abrupt discontinuation of a beta blocker can result in A. myocardial infarction B. hyperglycemia C. bradycardia D. hypotension

A. myocardial infarction The abrupt discontinuation of a beta blocker can produce angina pectoris or even myocardial infarction.

Hypothermia in the elderly is known to increase the risk for (select two) A. myocardial ischemia B. surgical wound evisceration C. coagulopathy D. surgical necrosis

A. myocardial ischemia C. coagulopathy Temperature regulation is impaired in elderly patients. Shivering can increase oxygen consumption and lead to hypoxia, acidosis, and cardiopulmonary complications rapidly in elderly patients. Drug metabolism can be impaired with hypothermia, as can platelet function, and the risk for infection is greater.

The identification card for a patient's pacemaker is VVI. This programming code indicates that the pacemaker A. paces and senses in the ventricle B. paces in the atria and senses in the ventricle C. inhibits firing if it senses an atrial depolarization D. it is triggered by atrial tachycardia

A. paces and senses in the ventricle The three letters correspond to what chamber is paced, what chamber is sensed, and whether the pacemaker is capable of inhibiting firing, triggering firing, or both. V stands for ventricle, A for atria, D for dual, 0 for neither, T for triggered, and I for inhibited. In this example, the pacemaker both senses and paces in the ventricle and is inhibited if it detects a natural ventricular depolarization.

Side effects of prolonged lithium use or lithium toxicity include A. polyuria B. paroxysmal atrial tachycardia C. leukopenia D. hyperreflexia

A. polyuria Common side effects of prolonged lithium use include sinus bradycardia, sinus node dysfunction, atrioventricular block, changes in the T wave morphology, and ventricular irritability. Leukocytosis is often present. A vasopressin-resistant diabetes insipidus can develop that produces polyuria. Lithium toxicity is associated with skeletal muscle weakness, widening of the QRS complex, hypotension, heart block, and seizures. This occurs when the serum lithium concentration is higher than 2 mEq/L.

The increased atrial pressures that can result from atrioventricular dys-synchrony can produce A. polyuria B. increased intracranial pressure C. hypertension D. increased renin secretion

A. polyuria Polyuria may result from SVT (or any other atrial tachycardia resulting in AV dys-synchrony) because of an increased secretion of atrial natriuretic peptide. Atrial natriuretic peptide is released as a result of the increased atrial pressures that occur when the atria contract against closed AV valves.

A patient with a pacemaker with a program code of VVI has a heart rate of 104 and you see pacer spikes on the ECG. You would most accurately assume that A. the pacemaker ventricular sensing is faulty B. the pacemaker atrial sensing is faulty C. the pacemaker is not designed to sense at all D. the pacemaker is not faulty

A. the pacemaker ventricular sensing is faulty This programming code indicates that this pacemaker is designed to pace and sense in the ventricle and inhibits its own activity if it senses a spontaneous depolarization. Because the spontaneous heart rate is higher than is normal for a pacemaker and pacemaker spikes are still visible, the pacemaker is obviously failing to sense the spontaneous beats.

Quinidine, a cinchona alkaloid, can produce cinchonism. What symptoms would be consistent with this? (select two) A. tinnitus B. blurry vision C. ataxia D. memory deficit

A. tinnitus B. blurry vision Salicylates and cinchona alkaloids can produce cinchonism which includes symptoms such as tinnitus, blurry vision, decreased hearing, and gastrointestinal upset.

You are preparing to induce general anesthesia in a patient who takes citalopram for depression. You know that citalopram places the patient at increased risk for A. torsades de pointes B. complete heart block C. atrial fibrillation D. Wolff-Parkinson-White syndrome

A. torsades de pointes Citalopram is an SSRI antidepressant that causes dose-dependent increases in the QT interval. Because of this, it can place the patient at increased risk for torsades de pointes.

A patient undergoing emergency surgery for traumatic injury is in hypovolemic shock. You discover he is on warfarin daily. In addition to crystalloids and packed red blood cells, the most important treatment options would be (select two) A. vitamin K B. platelets C. cryoprecipitate D. fresh frozen plasma

A. vitamin K D. fresh frozen plasma In order to restore the coagulation cascade back to normal, it would be prudent to administer vitamin K and/or fresh frozen plasma to counteract the anticoagulant effects of the warfarin.

What is the recommended dose of fresh frozen plasma for severely traumatized, bleeding, and coagulopathic patients? A. 5 mL/kg B. 10-15 mL/kg C. 20-25 mL/kg D. 30 mL/kg

B. 10-15 mL/kg Although additional FFP may be required, the recommended dose of thawed FFP to be administered immediately after the arrival of severely traumatized, hemorrhaging, coagulopathic individual is 10-15 mL/kg.

You are concerned about the postoperative fluid status of an elderly patient with a history of congestive heart failure. On what postoperative day would you expect third space fluid to mobilize into the vascular system? A. 1 B. 2 C. 3 D. 4

B. 2 Third-space fluid begins to mobilize into the vascular space on postoperative day two. It is important to evaluate the elderly patient for signs of fluid volume excess such as rales, crackles, tachypnea, and orthopnea.

With age, renal cortical mass decreases by as much as A. 15 percent B. 25 percent C. 35 percent D. 45 percent

B. 25 percent The mass of the renal cortex lowers by 20-25 percent with age. Also, by age 80 approximately half of the glomeruli are lost.

At what age does the MAC value start to decrease? A. 30 B. 40 C. 50 D. 60

B. 40 MAC decreases by about 6% percent per decade after the age of 40.

Minimum alveolar concentration (MAC) decreases by ____ percent per decade. A. 3 B. 6 C. 8 D. 10

B. 6 Minimum alveolar concentration (MAC) decreases by six percent per decade.

The decrease in total body water that occurs with aging is primarily due to A. A decrease in blood volume B. A decrease in muscle mass C. A decrease in cerebrospinal fluid D. A decrease in adipose tissue

B. A decrease in muscle mass Total body water decreases with age, primarily due to the decrease in cellular water from the loss of muscle mass and increase in adipose tissue.

With its decline in age, which neurotransmitter is noted for its connection to Alzheimer's disease? A. GABA B. Acetylcholine C. Dopamine D. Serotonin

B. Acetylcholine Acetylcholine levels normally decline slightly with age, but in Alzheimer's disease, they can decrease by as much as 90%.

What substance is synthesized and released by the adrenal cortex when stimulated by angiotensin II? A. Angiotensin-converting enzyme B. Aldosterone C. Cortisol D. Vasopressin

B. Aldosterone Angiotensin-converting enzyme converts angiotensin I to angiotensin II. Angiotensin II then triggers the adrenal cortex to produce aldosterone.

Which of the following is most likely to result in an exacerbation of the symptoms of multiple sclerosis following general anesthesia? A. Reversal of muscle relaxation with neostigmine B. An increase in body temperature of 1 degree Celsius C. A decrease in body temperature of 1 degree Celsius D. Administration of a volatile anesthetic

B. An increase in body temperature of 1 degree Celsius Although anesthesia drugs are not associated with exacerbations in the symptoms of multiple sclerosis, any increase in body temperature (even as little as 1 degree Celsius) can produce dramatic exacerbations. It is possible that the increase in body temperature results in complete conduction block in demyelinated nerves.

What heart sounds are consistent with mitral valve stenosis? (select two) A. A holosystolic murmur at the apex B. An opening snap in early diastole C. A rumbling diastolic murmur D. A midsystolic click

B. An opening snap in early diastole C. A rumbling diastolic murmur Mitral stenosis is associated with an 'opening snap' that occurs in early diastole and a rumbling diastolic murmur heard best at the axilla or apex. A holosytolic murmur at the apex that radiates to the axilla is associated with mitral valve regurgitation. Mitral valve prolapse is associated with a midsystolic click and a late systolic murmur.

Which of the following valvular lesions is most likely to result in eccentric left ventricular hypertrophy? A. Aortic stenosis B. Aortic regurgitation C. Mitral stenosis D. Tricuspid stenosis

B. Aortic regurgitation The hallmark characteristic of aortic regurgitation is eccentric left ventricular hypertrophy. Aortic stenosis is associated with the development of concentric left ventricular hypertrophy. Mitral stenosis and mitral regurgitation are both associated with overload of the left atrium with mitral stenosis often resulting in right ventricular hypertrophy as pressures are transmitted through the pulmonary vascular tree.

What hormone is released from the cardiac atria in response to increased atrial stretch and increases the renal excretion of sodium and water? A. Aldosterone B. Atrial natriuretic peptide C. Vasopressin D. Angiotensin

B. Atrial natriuretic peptide ANP is released from the cardiac atria due to increased atrial stretch. It exhibits vasodilatory effects and increases the renal excretion of sodium and water.

Which of the following are compensatory mechanisms that occur due to congestive heart failure? (select two) A. Preload decreases B. Circulating norepinephrine levels increase C. Concentric hypertrophy occurs D. Adrenergic receptors are down-regulated

B. Circulating norepinephrine levels increase D. Adrenergic receptors are down-regulated Increased preload and sympathetic tone, activation of the renin- angiotensin system, and ventricular hypertrophy are the compensatory mechanisms that occur in the presence of heart failure. Because of the chronically increased levels of circulating catecholamines associated with congestive heart failure, the response of adrenergic receptors is diminished (down- regulated). As heart failure ensues, the ejection fraction decreases. As the left ventricle dilates, it accommodates more volume. Thus, the same ejection fraction of an increased volume will still be a normal stroke volume. As venous congestion and ventricular dilatation continues, however, clinical deterioration will eventually occur.

Which of the following contribute to the prolonged elimination half-life of many drugs in geriatric patients? (select two) A. Decreased vascular volume B. Decreased renal blood flow C. Increased lipid stores D. Decreased alpha-1 acid glycoprotein levels

B. Decreased renal blood flow C. Increased lipid stores Geriatric patients have increased adipose stores, which prolongs the action of lipid-soluble drugs. They also have decreased hepatic and renal blood flow which can prolong the action of drugs dependent on the liver and kidneys for elimination. Although they have a decreased vascular volume, this correlates with a higher initial plasma concentration of a drug, not a prolonged elimination half-life. Alpha-1 acid glycoprotein levels normally are not affected by aging.

Which of the following radiographic changes are consistent with cor pulmonale? (select two) A. Narrowing of the pulmonary arteries B. Decreased vascular markings in the peripheral fields C. Decreased retrosternal space on the lateral film D. Decreased right ventricular size

B. Decreased vascular markings in the peripheral fields C. Decreased retrosternal space on the lateral film The chest x-ray often reveals increased width of the right pulmonary artery and decreased pulmonary vascular markings in the lung periphery. A late sign of cor pulmonale may be a decrease in the retrosternal space on the lateral film as right ventricular enlargement becomes evident.

Which of the following drugs should be avoided in patients with Parkinson's disease? A. Fentanyl B. Droperidol C. Ketorolac D. Desflurane

B. Droperidol Because Parkinson's disease results in decreased dopamine levels in the basal ganglia, butyrophenones such as droperidol, which can antagonize the effects of dopamine in the basal ganglia, can exacerbate symptoms.

Highly protein-bound drugs display a higher concentration of the free form of the drug in the elderly population. Why is this? A. Elderly patients have an increase in the pulmonary uptake of drugs B. Elderly patients have lower albumin levels C. Elderly patients have increased renal mass D. Elderly patients have increased skeletal muscle blood flow

B. Elderly patients have lower albumin levels Overall, plasma protein-binding is decreased in the elderly population. This is due to a decrease in circulating albumin levels. Alpha-1 acid glycoprotein levels, however are unchanged.

Manifestations of cor pulmonale include A. Systolic murmur B. Elevated pulmonary artery pressures C. Respiratory alkalosis D. Prominent V wave on the central venous pressure tracing

B. Elevated pulmonary artery pressures Cor pulmonale is a sequence of symptoms that originate with hypoxia due to pulmonary pathology, often COPD. The hypoxia results in hypoxic pulmonary vasoconstriction and elevated pulmonary artery pressures. In the face of pulmonary hypertension, the right ventricle can begin to fail resulting in prominent A waves on the central venous pressure, jugular venous distention, hepatosplenomegaly, peripheral edema and often a diastolic murmur due to incompetence of the pulmonary valve.

Which of the following would be the most appropriate anesthetic technique for a patient with multiple sclerosis? A. General anesthesia with succinylcholine B. Epidural anesthesia with bupivacaine C. Spinal anesthesia with ropivacaine D. Spinal anesthesia with tetracaine

B. Epidural anesthesia with bupivacaine General anesthesia is the most common choice for anesthesia, but succinylcholine can potentially result in a hyperkalemic response. There may also be a prolonged duration when using nondepolarizing agents. Spinal anesthesia is typically avoided as there is speculation that the lack of myelin predisposes the neurons to the effects of local anesthetic toxicity. Epidural anesthesia is considered less of a risk than spinal anesthesia.

Which agents are metabolized by monoamine oxidase? (select two) A. Ephedrine B. Epinephrine C. Dopamine D. Phenylephrine

B. Epinephrine C. Dopamine Monoamine oxidase is an enzyme that is primarily found on the outer membranes of mitochondria. Through oxidative deamination, it serves to inactivate serotonin, dopamine, epinephrine, and norepinephrine. The subtypes are MAO-A and MAO-B. MAO-A selectively deaminates epinephrine, norepinephrine, and serotonin, whereas MAO-B deaminates phenylethylamine.

Which beta-blocker is metabolized rapidly in the blood by an esterase located in the red blood cell cytoplasm? A. Atenolol B. Esmolol C. Metoprolol D. Labetolol

B. Esmolol Esmolol is metabolized quickly in the blood by an esterase found in the RBC cytoplasm.

You are preparing to induce general anesthesia in a patient taking propranolol for hypertension. Which drug would you anticipate to have substantially higher than normal plasma concentrations after administration? A. Isoflurane B. Fentanyl C. Chloroprocaine D. Propofol

B. Fentanyl Propranolol decreases the clearance of amide local anesthetics, but not ester anesthetics such as chloroprocaine. The pulmonary uptake of fentanyl, however, is substantially decreased in patients taking propranolol. As a result, plasma concentrations shortly after injection can be 2-4 times higher than normal.

Which anti-epileptic drug is least likely to produce aplastic anemia? A. Valproate B. Gabapentin C. Phenytoin D. Carbamazepine

B. Gabapentin Antiepileptic drugs are associated with anemias. The symptoms are typically mild, but can occasionally produce aplastic anemia. They are most commonly seen with valproate, carbamazepine, and phenytoin.

Which issues would you expect to see in a geriatric patient? (select two) A. Problems eliminating excess sodium B. Problems retaining sodium C. Decreased alpha-1 acid glycoprotein levels D. Increased serum albumin levels

B. Haloperidol Haloperidol is an antipsychotic agent that doesn't exhibit any significant alpha-adrenergic activity. The principal use of haloperidol is the treatment of delirium and agitation in the intensive care unit.

What is the most common cause of sudden cardiac death in young athletes? A. Ischemic heart disease B. Hypertrophic cardiomyopathy C. Cerebrovascular accident D. Pericardial tamponade

B. Hypertrophic cardiomyopathy Hypertrophic cardiomyopathy is the most common cause of sudden cardiac death in young athletes. Another potential cause in this patient class, however, is long QT syndrome, which has an incidence of 1 in 2,500 to 1 in 10,000.

Which of the following is a potential side effect of DDAVP administration in the treatment of von Willebrand disease? A. Prolongation of the PTT B. Hypotension C. Hepatic enzyme induction D. A decrease in factor IX concentration

B. Hypotension Too rapid administration of DDAVP IV can cause hypotension.

Which of the following interventions will not help decrease intracranial pressure? A. Ventriculostomy B. Hypoventilation C. Administration of mannitol D. 30 degree head-up position

B. Hypoventilation The most effective method to rapidly decrease intracranial pressure is via a lumbar cerebrospinal fluid drain or a ventriculostomy. Hyperventilation and the administration of drugs such as corticosteroids, diuretics, mannitol, and barbiturates also help decrease the intracranial pressure.

What is the most potent stimulus for pulmonary vasoconstriction? A. Hypotension B. Hypoxia C. Acidosis D. Hypocapnia

B. Hypoxia Generalized alveolar hypoxia is the most potent stimulus for pulmonary vasoconstriction. When it occurs locally, it serves to shunt blood to areas that are better oxygenated. Systemic acidosis also promotes pulmonary vasoconstriction but is not as strong a stimulus as hypoxia.

Select the two most dramatic changes in lung volumes seen with increased age. A. Increase in total lung capacity B. Increase in closing volume C. Increase in closing capacity D. Decrease in functional residual capacity

B. Increase in closing volume C. Increase in closing capacity Although decreases in total lung capacity and increases in the functional residual capacity are associated with aging, the most significant changes are the increases in both closing volume and closing capacity.

How does chronic hypertension affect the left ventricle? (select two) A. It can produce eccentric hypertrophy B. It can reduce left ventricular subendocardial perfusion C. It increases the left ventricular ejection fraction D. It increases left ventricular oxygen demand

B. It can reduce left ventricular subendocardial perfusion D. It increases left ventricular oxygen demand Faced with the increased afterload associated with chronic hypertension, the left ventricle increases its myocardial mass (concentric hypertrophy). As the left ventricle undergoes compensatory concentric hypertrophy, the increased myocardial mass has a higher oxygen demand while the contraction of the hypertrophied muscle can reduce subendocardial oxygen supply, placing the myocardium at greater risk for ischemia.

Which of the following statements regarding mitral valve stenosis is false? A. It is associated with a decreased left ventricular volume B. It is associated with a decreased left atrial pressure and increased right atrial pressure C. It can cause right ventricular failure D. It can result in pulmonary edema

B. It is associated with a decreased left atrial pressure and increased right atrial pressure As flow through the mitral valve opening into the left ventricle is decreased (thus decreasing the left ventricular volume), left atrial pressure increases, resulting in left atrial hypertrophy and distention. The increased pressure is transmitted into the pulmonary vasculature as the volume of pulmonary blood increases. This increased pulmonary vascular pressure represents an increase in right ventricular afterload and will cause right ventricular hypertrophy and failure. As pulmonary venous pressure increases above about 25 mmHg, fluid can leak into the pulmonary interstitial space resulting in a decrease in pulmonary compliance and increased work of breathing. If the change in pulmonary venous pressure occurs over a long period of time, an increase in pulmonary lymph flow can partially compensate for the fluid accumulation.

Ropivacaine is better suited than bupivacaine for a lumbar epidural for a laboring parturient because (select two) A. It has a longer half-life than bupivacaine B. It produces less motor block than bupivacaine C. It causes less central nervous system toxicity than bupivacaine D. It requires half of the volume to achieve the same clinical effect

B. It produces less motor block than bupivacaine C. It causes less central nervous system toxicity than bupivacaine Ropivacaine produces less motor block than bupivacaine, causes less CNS and cardiac toxicity, and has an elimination half-life of 5.2 hours compared to 10.9 hours for bupivacaine.

An obstruction of which coronary artery is associated with the least favorable prognosis? A. Right coronary artery B. Left main coronary artery C. Circumflex artery D. Left coronary artery

B. Left main coronary artery Obstruction of the left main coronary artery is considered the most serious cardiac vessel lesion and has the least favorable prognosis despite medical treatment.

What is the most common cause of right ventricular heart failure? A. Pulmonary hypertension B. Left ventricular heart failure C. Myocardial infarction D. Endocarditis

B. Left ventricular heart failure The most common cause of right ventricular failure is left ventricular failure. Causes of isolated right ventricular failure include pulmonary hypertension or right ventricular myocardial infarction.

Which of the following statements does not correspond with an accurate understanding of the appropriate anesthetic management goals for the patient with aortic stenosis? A. Normal sinus rhythm should be maintained B. Myocardial depression reduces the severity of the lesion C. Hypotension should be treated aggressively D. Both bradycardia and tachycardia should be avoided

B. Myocardial depression reduces the severity of the lesion Because left ventricular filling is so dependent upon an appropriately-timed atrial contraction, maintenance of normal sinus rhythm is very important. The development of a junctional rhythm or atrial fibrillation can result in congestive heart failure and hypotension. Hypotension must be treated aggressively as the corresponding decrease in myocardial perfusion can decrease left ventricular function and result in a decrease in cardiac output which would further worsen hypotension. Sustained increases in heart rate must be avoided as the ventricles need adequate time for ventricular filling. Conversely, severe bradycardia can result in overfilling and ventricular distention.

Which of the following statements regarding multiple sclerosis is not true? A. It is more common in female patients B. Patients with multiple sclerosis are more likely to relapse during the third trimester of pregnancy C. It is characterized by demyelination of axons D. Exposure to viral and bacterial infections can trigger relapses

B. Patients with multiple sclerosis are more likely to relapse during the third trimester of pregnancy Multiple sclerosis is twice as likely to occur in women as in men, it is characterized by progressive demyelination of axons, and bacterial and viral infections can trigger relapses; however, the relapse rate diminishes during pregnancy, especially during the third trimester. The relapse rate increases during the first three months postpartum.

Which classes of patients are most at risk for intracranial hemorrhage if thrombolytic therapy is administered in the setting of acute myocardial infarction? (select two) A. Patients with hydrocephalus B. Patients with uncontrolled hypertension C. Patients with a first-degree relative diagnosed with a cerebral aneurysm D. Patients over the age of 75

B. Patients with uncontrolled hypertension D. Patients over the age of 75 Patients older than 75 and patients with uncontrolled hypertension are most at risk for intracranial hemorrhage from the administration of thrombolytic therapy.

Which of the following changes are expected to occur in the elderly as part of the normal aging process? (select two) A. Hepatic microsomal activity is decreased B. Pulmonary collagen content increases C. Renal blood flow decreases D. The alveolar-arterial difference for oxygen decreases

B. Pulmonary collagen content increases C. Renal blood flow decreases Although hepatic and renal blood flow both decrease, hepatic microsomal enzyme activity is generally well preserved in the elderly. The effect of aging on the pulmonary system is a decrease in elastic tissue and an increase in the amount of collagen resulting in a 15% reduction in the functional alveolar surface area. The alveolar-arterial difference for oxygen increases from approximately 8 mm Hg at age 20 to approximately 20 mm Hg at age 70.

Which statements concerning myocardial ischemia and changes in the ST segment on the ECG are correct? (select two) A. ST segment elevation is typically associated with subendocardial ischemia B. ST segment depression is typically associated with subendocardial ischemia C. ST segment elevation is typically associated with transmural ischemia D. ST segment depression is typically associated with transmural ischemia

B. ST segment depression is typically associated with subendocardial ischemia C. ST segment elevation is typically associated with transmural ischemia Depression of the ST segment is typically associated with subendocardial ischemia. ST segment elevation is typically associated with transmural ischemia.

An increase in blood pressure due to coarctation of the aorta would be an example of A. Essential hypertension B. Secondary hypertension C. Pre-hypertension D. Hypertensive emergency

B. Secondary hypertension Secondary hypertension is an increase in blood pressure due to a cause that can be identified and cured such as pheochromocytoma, renal artery stenosis, coarctation of the aorta, Conn's syndrome, or Cushing's disease. Essential hypertension is an increased blood pressure for which there is no identifiable cause and is diagnosed based on the exclusion of causes such as those listed above. Approximately 95% of patients with hypertension are diagnosed with essential hypertension. Prehypertension is defined as a systolic blood pressure between 120-139 or a diastolic blood pressure between 80-89. A diastolic pressure higher than 109 mmHg with signs of end-organ damage is designated as a hypertensive emergency. A diastolic pressure higher than 109 mmHg in the parturient is designated as hypertensive emergency even if no signs of end-organ damage are present.

The use of nitrates in the treatment of an acute myocardial infarction is contraindicated in combination with which of the following drugs? A. Verapamil B. Sildenafil C. Clopidogrel D. Acebutolol

B. Sildenafil Nitrates are contraindicated in patients who have taken sildenafil within the previous 24 hours because of risk of severe hypotension. Verapamil is a calcium channel blocker, clopidogrel is an adenosine diphosphate inhibitor, and acebutolol is a beta blocker. All three of the drugs are adjunct medications used in the treatment and prevention of angina.

What are the safest neuromuscular blocking agents to use in a patient in liver failure? (select two) A. Pancuronium B. Succinylcholine C. Rocuronium D. Cisatracurium

B. Succinylcholine D. Cisatracurium Cisatracurium and succinylcholine are the safest agents to use in a patient with liver failure because they undergo no liver metabolism.

Which of the following statements describes the age-related changes in glomerular function accurately? A. Dosing adjustments based on renal function do not need to be made until about age 70 B. The glomerular filtration rate decreases by 1 mL/min each year after age 40 C. By age 80, over 75% of the glomeruli are lost D. The glomerular filtration rate does not appreciably change with age

B. The glomerular filtration rate decreases by 1 mL/min each year after age 40 The glomerular filtration rate decreases about 1 mL/min each year after age 40. This can reduce renal excretion of drugs to a level where dosing adjustments need to be made by age 60. By age 80, over half of the glomeruli are lost.

Which of the following statements is true of the pharmacokinetic changes seen in the elderly population? A. They have a higher volume of distribution for succinylcholine B. They have a lower volume of distribution for glycopyrrolate C. They have a lower volume of distribution for thiopental D. They have a lower volume of distribution for midazolam

B. They have a lower volume of distribution for glycopyrrolate In the elderly patient, total body water decreases while total body fat increases. Thus, the volume of distribution for water-soluble drugs such as glycopyrrolate, succinylcholine, and gentamicin decreases while the volume of distribution for lipid-soluble drugs such as barbiturates, benzodiazepines, and volatile anesthetics increases.

The elimination of hydrophilic agents in the elderly is prolonged primarily because of A. a decrease in hepatic blood flow B. a decrease in renal clearance C. a decrease in volume of distribution D. a decrease in serum pH

B. a decrease in renal clearance The elimination of hydrophilic agents is prolonged in the elderly primarily due to a decrease glomerular filtration rate and decreased renal blood flow.

Transfusion-related acute lung injury is more likely to occur when the donated blood is from A. a male donor B. a female donor C. a donor with myasthenia gravis D. a donor with polycythemia

B. a female donor TRALI is most commonly due to human leukocyte antigen or human neutrophil antigen from the donor. Because these antigens are more common in females, the risk of TRALI is more common when using donor blood from females.

Which of the following side effects is most likely to occur with the use of intraoperative blood salvage (cell saver)? A. hyponatremia B. air embolism C. hyperkalemia D. fluid volume overload

B. air embolism The most common side effects of intraoperative blood salvage are air embolism, abnormalities in coagulation, and DIC-like syndrome (also known as salvaged-blood syndrome).

A patient with congestive heart failure exhibits frequent episodes of malignant ventricular ectopy. He is not currently a candidate for the placement of an AICD. What drug would be most appropriate for use in this patient for the treatment of the arrhythmias? A. verapamil B. amiodarone C. lidocaine D. aspirin

B. amiodarone Amiodarone reduces the risk of sudden cardiac death by 29% in patients with congestive heart failure. Therefore, it is the best alternative for patients who refuse or are not candidates for an AICD.

What is the most significant anesthetic concern for the patient with Huntington's chorea? A. risks associated with aortic stenosis B. aspiration pneumonia C. resistance to succinylcholine D. strict avoidance of phenothiazines

B. aspiration pneumonia Huntington's chorea is characterized by choreiform movements, athetosis, and dystonia resulting from genetically-linked neuronal atrophy. As the disease progresses, weakness of the pharyngeal muscles predisposes the patient to aspiration pneumonia, a common cause of death. Drugs that reduce dopaminergic transmission such as thioxanthines, butyrophenones, and phenothiazines reduce the severity of chorea. Decreased plasma cholinesterase may prolong the effect of succinylcholine. There are no specific contraindications to the use of any inhaled or intravenous anesthetics with Huntington's chorea.

In the extrinsic coagulation pathway, disruption of the endothelium leads to exposure of tissue factor which binds to A. prothrombin B. factor VII C. calcium D. factor X

B. factor VII The extrinsic pathway is initiated by the release of a group of proteins known as Tissue Factor when tissues are damaged. Tissue Factor forms a complex with Factor VII, and in an enzymatic reaction requiring calcium, catalyzes the activation of Factor X.

What ECG alterations would you most likely see in patients taking tricyclic antidepressants? (select two) A. shortening of the PR interval B. flattening of the T wave C. widening of the QRS D. atrial flutter

B. flattening of the T wave C. widening of the QRS Tricyclic antidepressants depress cardiac conduction through the atria and ventricles resulting in prolongation of the PR interval, widening of the QRS, and flattening or inversion of the T wave.

A patient with mitral stenosis has a left atrial pressure of 27 mmHg. You would presume that the patient likely A. is asymptomatic B. has pulmonary hypertension C. has decreased preload D. would benefit from a balloon aortic valvotomy

B. has pulmonary hypertension A mean left atrial pressure of 25 mmHg is required when the mitral area is less than 1 cm2. If this pressure is sustained, the patient will likely develop pulmonary hypertension.

What is the most frequent cause of hospitalization in patients older than 65? A. hip fracture B. heart failure C. renal failure D. ketoacidosis

B. heart failure The most frequent cause of hospitalization in patients older than 65 is heart failure.

Which side effect would you most expect to see after administration of protamine? A. bradycardia B. hypotension C. thrombocytopenia D. transient blindness

B. hypotension Protamine has been noted to produce adverse side effects such as anaphylaxis, pulmonary vasoconstriction, right ventricular failure, and hypotension.

The antiarrhythmic mexilitine is electrophysiologically most similar to A. quinidine B. lidocaine C. bretylium D. verapamil

B. lidocaine The antiarrhythmic mexilitine is an orally-administered analogue of lidocaine. It is used for the chronic treatment of ventricular arrhythmias. Electrophysiologically, it is most similar to lidocaine.

Sildenafil, tadalafil, and vardenafil inhibit A. nitric oxide B. phosphodiesterase Type 5 (PDE5) C. adenosine triphosphate (ATP) D. ionized calcium

B. phosphodiesterase Type 5 (PDE5) Sildenafil, tadalfil, and vardenafil inhibit cyclic GMP (cGMP)-specific phosphodiasterase type-5 (PDE5) in vascular smooth muscle. Blocking PDE5 stops cGMP breakdown.

Adenosine would be useful in the treatment of A. ventricular arrhythmias B. supraventricular tachycardia C. atrial fibrillation D. atrial flutter

B. supraventricular tachycardia Adenosine slows the conduction rate through the AV node. This makes it useful in the treatment of supraventricular tachycardias. It is not useful, however, in treating atrial fibrillation, atrial flutter, or ventricular tachycardias.

A patient taking MAO inhibitors exhibits hypotension under general anesthesia. The recommended pharmacologic strategy to address the hypotension would be to administer A. the normal dose of ephedrine B. twice the normal dose of ephedrine C. 1/3 the usual dose of phenylephrine D. three times the normal dose of epinephrine

C. 1/3 the usual dose of phenylephrine Patients taking MAO inhibitors can have dramatic responses to indirect-acting sympathomimetics such as ephedrine. The recommended approach is to treat hypotension with a direct-acting agent such as phenylephrine. It is important to note, however, that patients taking these drugs can have an increased response to direct-acting agents as well. Therefore, you should begin with 1/3 the normal dose of a direct-acting agent and then titrate the dose to effect.

In a patient who is 65 years of age, you would expect that the mean alveolar concentration for an inhaled anesthetic would be decreased by about A. 5% B. 10% C. 15% D. 20%

C. 15% The MAC for inhaled anesthetics decreases by about 6% for each decade after the age of 40. This patient is 2.5 decades past 40, which would correspond with a 15% decrease in MAC.

What is the normal aortic valve area? A. 1.5-2.0 square centimeters B. 2.0-2.5 square centimeters C. 2.5-3.5 square centimeters D. 4.5-5.5 square centimeters

C. 2.5-3.5 square centimeters The normal aortic valve area is 2.5 - 3.5 cm2.

When inducing general anesthesia for a patient with chronic hypertension, you should strive to maintain the mean arterial blood pressure within what percent of normal? A. 10 B. 15 C. 20 D. 30

C. 20 When inducing general anesthesia for a patient with chronic hypertension, you should strive to maintain the mean arterial blood pressure within 20 percent of normal.

Pulmonary hypertension is defined as a mean pulmonary artery pressure above _____ when measured during right heart catheterization. A. 15 mmHg B. 20 mmHg C. 25 mmHg D. 35 mmHg

C. 25 mmHg Pulmonary arterial hypertension exists when the mean pulmonary artery pressure is greater than 25 mmHg.

You would expect serum troponin levels to become elevated within how many hours following the onset of an acute myocardial infarction? A. 1 hour B. 2 hours C. 3 hours D. 8 hours

C. 3 hours Troponin is a cardiac-specific marker for myocardial infarction that becomes elevated within 3 hours of myocardial injury and remains elevated for several days following the event.

What intravenous dose of amiodarone is recommended in patients with ventricular tachycardia that is resistant to electrical defibrillation? A. 30 mg B. 100 mg C. 300 mg D. 1000 mg

C. 300 mg Amiodarone 300 mg IV is recommended for patients with atrial fibrillation and ventricular tachycardia that is resistant to electrical defibrillation.

With aging, the ventilatory response to hypercapnia can decrease by as much as _____ percent. A. 10 B. 30 C. 50 D. 70

C. 50 With aging, the ventilatory response to hypercapnia decreases by as much as 50 percent.

Which of the following statements concerning the effects of muscle relaxants in the elderly is not true? A. The onset of all muscle relaxants is prolonged B. The elimination half-life of atracurium is not affected by age C. A decrease in the rate of hepatic elimination results in a prolonged elimination half-life of pancuronium D. Elderly patients have a longer circulatory time

C. A decrease in the rate of hepatic elimination results in a prolonged elimination half-life of pancuronium Because of a longer circulatory time and decrease in skeletal muscle blood flow, the onset of all muscle relaxants is up to twice as long in elderly patients. The elimination half-life of atracurium is not affected substantially by age. The elimination of pancuronium is prolonged in the elderly, but since it is eliminated primarily via the renal route, it is the decrease in renal clearance, not hepatic clearance that prolongs pancuronium.

Which of the following agents produces irreversible inhibition of platelet function by preventing the synthesis of thromboxane A2? A. Phosphodiesterase inhibitors B. ADP receptor antagonists C. Acetylsalicylic acid D. Glycoprotein Iib/IIIa receptor antagonists

C. Acetylsalicylic acid Cyclooxygenase inhibitors such as aspirin cause irreversible inhibition of platelet cyclooxygenase which prevents synthesis of thromboxane A2. ADP receptor antagonists such as ticlopidine and clopidogrel block the ADP receptor non-competitively and irreversibly inhibit IIb/IIIa platelet aggregation. Phosphodiesterase inhibitors such as dipyridamole act by inhibiting phosphodiesterase which results in an increase in cyclic AMP which is an inhibitor of platelet aggregation. Glycoprotein Iib/IIIa receptor antagonists such as abciximab reversibly block the Iib/IIIa site by which fibrinogen crosslinks platelets together.

What is the most effective means of preventing the postinduction hypotension commonly seen in hypertensive patients? A. Administration of ephedrine prior to induction B. Administration of neosynephrine after induction C. Administration of crystalloids prior to induction D. Administration of dopamine during induction

C. Administration of crystalloids prior to induction Because the chronically hypertensive patient is volume contracted, adequate hydration prior to induction will usually attenuate the drop in blood pressure seen after induction of anesthesia.

What is the initial compensatory mechanism as aortic stenosis develops? A. An increase in afterload B. An increase in right ventricular pressure C. An increase in left ventricular pressure D. A decrease in pulmonary vascular resistance

C. An increase in left ventricular pressure The initial compensatory mechanism for aortic stenosis is an increase in left ventricular pressure to maintain flow through the narrowed aortic valve. This results in a pressure gradient across the valve.

Which of the following statements regarding aortic stenosis is true? A. Aortic stenosis is associated with an increase in left ventricular compliance B. Myocardial oxygen demand is not altered in aortic stenosis C. Aortic stenosis is associated with diastolic dysfunction D. Aortic stenosis does not alter arterial flow to the myocardium

C. Aortic stenosis is associated with diastolic dysfunction In aortic stenosis, left ventricular compliance decreases as the left ventricle hypertrophies, resulting in diastolic dysfunction. The myocardial demand is increased due to ventricular hypertrophy and the supply is decreased as the extraordinary compression of intramyocardial vessels during systole restricts arterial flow to the myocardium.

Which of the following medications would you expect a patient with Alzheimer's disease to be taking to slow the rate of cognitive deterioration? A. Anticholinesterase inhibitors B. Anticholinergics C. Cholinesterase inhibitors D. Antimuscarinics

C. Cholinesterase inhibitors Patients with Alzheimer's disease are often prescribed cholinesterase inhibitors such as tacrine, donepezil, galantamine, and rivastigmine to slow the progression of cognitive deterioration. Anticholinesterase drugs are utilized to reverse nondepolarizing muscle relaxant blockade. Anticholinergics and antimuscarinics refer to the same class of drugs that block the effects of acetylcholine. Anticholinesterase inhibitor is not a true class of drugs and is only presented as a distractor due to its similarity to acetylcholinesterase inhibitor (another term for cholinesterase inhibitor).

Which of the following is not decreased in elderly patients? A. Aldosterone secretion B. Blood volume C. Circulatory time D. Glomerular filtration rate

C. Circulatory time The circulatory time is increased in the elderly. As a result, intravenous induction will progress more slowly as it takes longer for the administered drugs to reach their target sites. The glomerular filtration rate decreases about 1 mL/min each year after age 40. Blood volume decreases with age. In elderly patients, the kidneys are not as effective at eliminating excess sodium, nor are they as effective at retaining sodium. This is due primarily to a reduction in the secretion of aldosterone.

Which of the following antiplatelet drugs used in patients with a history of myocardial ischemia works by blocking adenosine diphosphate receptors? A. Abciximab B. Eptifibatide C. Clopidogrel D. Aspirin

C. Clopidogrel Clopidogrel and ticlopidine inhibit platelet aggregation by blocking adenosine diphosphate receptors. Abciximab and eptifibatide inhibit platelet adhesion, activation, and aggregation by antagonizing platelet glycoprotein IIa/IIIb receptors.

The increase in peripheral vascular resistance seen in elderly patients typically results in A. Hypotension B. Eccentric left ventricular hypertrophy C. Concentric left ventricular hypertrophy D. Increased cardiac output

C. Concentric left ventricular hypertrophy The increase in peripheral vascular resistance (afterload) in the elderly occurs primarily due to progressive loss of arterial distensibility and leads to concentric left ventricular hypertrophy. The cardiac output may decrease due to myocardial fiber atrophy, but as the increase in afterload is greater than the decrease in cardiac output, hypertension usually results.

On preoperative examination of a patient with Parkinson's disease, you find that he is taking levodopa. Considering that you are planning a general anesthetic, you should A. Discontinue the levodopa 24 hours prior to surgery B. Discontinue the levodopa 12 hours prior to surgery C. Continue the levodopa as scheduled D. Increase the dose of levodopa

C. Continue the levodopa as scheduled Levodopa has a relatively short half-life and interruption of administration of the drug can result in abrupt loss of therapeutic effect within 6 to 12 hours resulting in skeletal muscle rigidity which can interfere with ventilation. Therefore, it is recommended that levodopa be continued as prescribed.

Which of the following tests of renal function would you expect to be unchanged by advanced age? A. Glomerular filtration rate B. Renal blood flow C. Creatinine D. Creatinine clearance

C. Creatinine Due primarily to fibrotic changes and decreased mass in the renal cortex, glomerular filtration rate and creatinine clearance decrease with age. Even though creatinine clearance decreases, the reduced muscle mass and creatinine production associated with increased age result in an unchanged serum creatinine level.

Which of the following statements regarding intraocular pressure is true? A. An intraocular pressure of 15 mmHg is considered abnormal B. Intraocular pressure is lower in the morning C. Dextran decreases intraocular pressure D. Sorbitol increases intraocular pressure

C. Dextran decreases intraocular pressure The normal intraocular pressure is about 10 to 21 mmHg and pressures above 22mmHg is considered intraocular hypertension. Intraocular pressure is higher in the morning, presumably from venous congestion due to the eyes being closed. Dextran, mannitol, urea, and sorbitol all decrease intraocular pressure by exerting a hyperosmotic effect that decreases aqueous humor formation.

Which of the following drugs would not exhibit a smaller volume of distribution in the geriatric patient? A. Glycopyrrolate B. Succinylcholine C. Diazepam D. Gentamicin

C. Diazepam In the elderly patient, total body water decreases while total body fat increases. Thus, the volume of distribution for water-soluble drugs such as glycopyrrolate, succinylcholine, and gentamicin decreases while the volume of distribution for lipid-soluble drugs such as barbiturates, benzodiazepines, and volatile anesthetics increases.

What of the following would decrease the amount of prolapse in patients with mitral valve prolapse? (select two) A. Increased ejection fraction B. Vasodilation C. Increased intravascular volume D. Hypertension

C. Increased intravascular volume D. Hypertension Any factor that maintains a larger ventricular volume will decrease the degree of prolapse. Hypertension, vasoconstriction, drug-induced myocardial depression, and increased preload will decrease the degree of prolapse.

All of the following are potential reasons for delayed recovery from anesthesia in geriatric patients except: A. Decreased renal function B. Decreased hepatic function C. Increased protein binding D. Increased body fat percentage

C. Increased protein binding Decreased renal function and decreased hepatic function are normal findings in aged patients and can both contribute to a prolonged effect in many drugs. Body fat can act as a reservoir for lipid-soluble drugs which can result in prolongation of their action. Increased protein binding would not necessarily result in a prolonged duration of action of a a drug and furthermore, increased serum protein is not a typical finding in the geriatric population as protein synthesis also diminishes with age.

Which of the following hemodynamic patterns is consistent with a diagnosis of sepsis? A. High pulmonary capillary wedge pressure, high cardiac output, low systemic vascular resistance B. Low pulmonary capillary wedge pressure, Low cardiac output, high systemic vascular resistance C. Low pulmonary capillary wedge pressure, high cardiac output, low systemic vascular resistance D. High pulmonary capillary wedge pressure, high cardiac output, high systemic vascular resistance

C. Low pulmonary capillary wedge pressure, high cardiac output, low systemic vascular resistance

A young, otherwise healthy athlete has long QT syndrome. During anesthesia, he exhibits torsades de pointes. What is the treatment of choice for this? A. Amiodarone B. Verapamil C. Magnesium D. Lidocaine

C. Magnesium Long QT syndrome has an incidence between 1 in 2,500 and 1 in 10,000. During anesthesia for a patient with this disorder, it is important to avoid sympathetic stimulation and to keep the heart rate less than 130 BPM. Torsades de pointes may develop under anesthesia in these patients. It is often short-lived and self-terminating, but the treatment of choice for it is magnesium.

Which drugs are selective beta-1 adrenergic blockers? A. Metoprolol, labetalol, esmolol B. Atenolol, propranolol, metoprolol C. Metoprolol, atenolol, esmolol D. Timolol, atenolol, sotalol

C. Metoprolol, atenolol, esmolol Metoprolol, atenolol, and esmolol are selective beta-1 adrenergic blockers.

Which test assesses the integrity of the extrinsic hemostasis pathway? A. Platelet count B. Partial thromboplastin time C. Prothrombin time D. Bleeding time

C. Prothrombin time The PT assesses the effectiveness of the extrinsic hemostasis pathway.

What is the most common cause of secondary hypertension? A. Obesity B. Hyperlipidemia C. Renal artery stenosis D. Hyperaldosteronism

C. Renal artery stenosis Secondary hypertension is an increase in blood pressure due to a cause that can be identified and cured such as pheochromocytoma, renal artery stenosis, coarctation of the aorta, Conn's syndrome, or Cushing's disease. Renal artery stenosis is the most common cause of secondary hypertension.

Which of the following agents would be more likely to result in a dangerous increase in intraocular pressure in the patient with glaucoma? A. Atropine B. Glycopyrrolate C. Scopolamine D. These agents exert equivalent effects

C. Scopolamine The amount of atropine typically administered preoperatively (0.4 mg IV) results in approximately 0.0001 mg absorbed by the eye which many practitioners regard as having an insignificant effect on intraocular pressure. Scopolamine has a greater mydriatic effect than atropine and is not recommended for use in patients with glaucoma.

What three factors determine the regurgitant volume in mitral regurgitation? (select two) A. Pulmonary vascular resistance B. Right atrial pressure C. Size of the mitral valve opening D. The pressure gradient across the mitral valve

C. Size of the mitral valve opening D. The pressure gradient across the mitral valve The size of the mitral valve opening, the heart rate, and the pressure gradient across the valve determine the regurgitant volume in mitral regurgitation.

Which of the following is true regarding cerebrovascular accident or stroke? A. Most strokes are hemorrhagic in nature B. Ischemic strokes are four times more lethal than hemorrhagic strokes C. The incidence of stroke is the same in men and women over the age of 75 D. Dextrose is commonly administered to protect neural function in an ischemic stroke

C. The incidence of stroke is the same in men and women over the age of 75 Most CVAs are ischemic in nature. Only about 10 percent are hemorrhagic. Hemorrhagic strokes are four times more lethal than ischemic strokes. The risk for a stroke is higher in men until age 75 when the risk for men and women is the same. Hyperglycemia is associated with poor outcomes in patients with ischemic stroke.

The extrinsic pathway of the hemostatic response is initiated by the release of a group of proteins known as A. Proaccelerins B. Fibrinogen C. Tissue factor D. Collagen

C. Tissue factor The extrinsic pathway is initiated by the release of a group of proteins known as Tissue Factor when tissues are damaged. Tissue Factor forms a complex with Factor VII, and in an enzymatic reaction requiring calcium, catalyzes the activation of Factor X. Proaccelerin is the other name for factor V, and fibrinogen is another name for factor I. Exposure of the blood to collagen can initiate the intrinsic pathway.

All of the lung parameters below increase with age except for A. Closing capacity B. Functional residual capacity C. Total lung capacity D. Residual volume

C. Total lung capacity The total lung capacity doesn't change significantly due to age alone. Functional residual capacity increases due to an increase in residual volume. The closing volume and closing capacity eventually increase to such an extent, however, that they exceed functional residual capacity.

Which anesthetic management goals are common for patients with mitral regurgitation and patients with mitral stenosis? (select two) A. You should maintain a decreased cardiac contractility in both disorders B. You should maintain a normal to increased heart rate in both disorders C. You should avoid increases in pulmonary vascular resistance in both disorders D. Preload should be maintained at a normal level for both disorders

C. You should avoid increases in pulmonary vascular resistance in both disorders D. Preload should be maintained at a normal level for both disorders In mitral regurgitation, your goal is to maintain an increased heart rate and decreased afterload, while the goal in mitral stenosis is to maintain a normal or decreased heart rate and normal afterload. In both disorders, you should maintain normal sinus rhythm, avoid increases in pulmonary vascular resistance, and maintain preload at normal to increased levels.

Elderly patients have A. decreased sympathetic activity at rest B. a decreased systolic blood pressure C. a diminished cardiac response to beta-receptor stimulation D. an increase in the number of sinus node pacemaker cells

C. a diminished cardiac response to beta-receptor stimulation Elderly patients have a diminished cardiac response to beta-receptor stimulation and increased sympathetic activity at rest. The number of sinus node cells decreases with age.

The geriatric population is more susceptible to decreases in core temperature primarily because A. basal metabolic rate increases with age B. decreased body fat results in increased heat loss C. autonomic peripheral vasoconstriction decreases with age D. they exhibit increased lean body mass

C. autonomic peripheral vasoconstriction decreases with age The elderly population is more susceptible to decreases in core temperature primarily due to a decrease in basal metabolic rate and decreased autonomic peripheral vasoconstriction. Elderly patients have an increased body fat reserve.

Which of the following agents is a low molecular weight heparin? A. warfarin B. ximelagatran C. dalteparin D. rivaroxaban

C. dalteparin Dalteparin and enoxaparin are low molecular weight heparins that exhibit higher anti-Xa and anti-IIa activity than regular heparin. Warfarin is a vitamin K antagonist. Rivaroxaban and apixaban are direct factor Xa inhibitors. Ximelagatran is a direct thrombin inhibitor.

What primarily explains the prolonged duration of midazolam in an elderly patient? A. decreased renal perfusion B. decreased protein binding C. decreased hepatic perfusion D. increased absorption into the muscle tissue

C. decreased hepatic perfusion The duration of action of midazolam is increased in the elderly. The clearance of the drug is reduced by as much as 30 percent due to a decrease in hepatic mass and perfusion. The recommended dose for a 90 year-old should be reduced by 75 percent from that acceptable for a 20 year-old.

Hemophilia B can be treated with A. factor VII concentrate B. factor VIII concentrate C. factor IX concentrate D. desmopressin and cryoprecipitate

C. factor IX concentrate Hemophilia B (also called Christmas disease) is treated with factor IX concentrate. Hemophilia A can be treated with desmopressin, cryoprecipitate, and factor VIII concentrate. Congenital factor VII deficiency is known as Alexander's disease or proconvertin deficiency. Severe liver disease is also associated with an acquired factor VII deficiency.

The most common serious side effect of MAO inhibitors is A. sinus bradycardia B. ventricular fibrillation C. orthostatic hypotension D. hepatitis

C. orthostatic hypotension The most common serious side effect of MAO inhibitors is orthostatic hypotension. The effect is especially prominent in elderly patients.

Which test is most widely used to assess the intrinsic coagulation pathway? A. platelet level B. prothrombin time C. partial thromboplastin time D. INR

C. partial thromboplastin time The partial thromboplastin time is a commonly used test that assesses the intrinsic coagulation pathway. The prothrombin time evaluates the extrinsic pathway. The INR is a normalization calculation performed on the prothrombin time.

Warfarin therapy is best monitored with the A. bleeding time B. partial thromboplastin time C. prothrombin time D. thrombin time

C. prothrombin time The prothrombin time is especially sensitive to the vitamin K-dependent clotting factors and is best for monitoring warfarin therapy.

A patient you are preparing to take to surgery states he takes buproprion for smoking cessation. Compared to other antidepressant class medications, you know that buproprion exhibits an increased incidence of A. serotonin syndrome B. anticholinergic crisis C. seizures D. orthostatic hypotension

C. seizures Buproprion is structurally similar to amphetamines and is used in the treatment of depression and smoking cessation. Like the SSRIs, it doesn't exhibit any anticholinergic effects, but does exhibit an increased risk of seizures (0.4%). Buproprion is not associated with orthostatic hypotension.

Methods to decrease the intracranial pressure should be employed when there is a sustained increase in pressure to at least A. 10 cm H2O B. 10 mm Hg C. 20 cm H2O D. 20 mm Hg

D. 20 mm Hg It is recommended that methods to decrease the intracranial pressure (head elevation, CSF drainage, diuretics, steroids, etc) be employed when there is a sustained increase in pressure of 20 mmHg or more.

In a patient with mitral valve regurgitation, corrective surgery is preferred to be performed when the ejection fraction is A. 30% B. 40% C. 50% D. >60%

D. >60% Corrective surgery for mitral regurgitation should be done when the ejection fraction is greater than 60%. If the patient is symptomatic, it should be performed even if the ejection fraction is normal. Otherwise, significant cardiac remodeling occurs that prevents any effective improvement in performance.

Which of the following would be an appropriate preoperative treatment for the patient with hemophilia A about to undergo surgery? A. Administration of recombinant factor IX B. Administration of factor IX-PCC C. Administration of platelet concentrate D. Administration of factor VIII concentrate

D. Administration of factor VIII concentrate Factor VIII concentrate should be administered to the patient with hemophilia A until levels are at or near 100% of normal prior to surgery, which may require a dose of 50-60 units/kilogram. Factor IX preparations are for use in patients with hemophilia B.

The presence of a bicuspid aortic valve predisposes the patient to the development of A. Tricuspid regurgitation B. Mitral regurgitation C. Eccentric left ventricular hypertrophy D. Aortic stenosis

D. Aortic stenosis Aortic stenosis is the narrowing of the aortic valve which results in obstruction of blood flow into the aorta. It is the result of degeneration and calcification of the leaflets of the aortic valve or the presence of a bicuspid rather than a normal tricuspid valve.

Which of the following does not decrease significantly with age? A. Lean muscle mass B. Bone density C. The number of functioning nephrons D. Baseline hepatic function

D. Baseline hepatic function By age 80, the number of functioning nephrons has decreased by 50 percent. Lean muscle mass decreases with age. Bone density decreases significantly with age, increasing the risk for fractures. Although hepatic mass decreases significantly, baseline hepatic function remains well preserved with age.

Which of the following echocardiographic findings would be consistent with aortic stenosis? A. Eccentric hypertrophy of the right ventricle B. Concentric hypertrophy of the right ventricle C. Eccentric hypertrophy of the left ventricle D. Concentric hypertrophy of the left ventricle

D. Concentric hypertrophy of the left ventricle The exposure of the left ventricle to the increased pressure gradients associated with aortic stenosis result in concentric hypertrophy of the left ventricle (thickening of the ventricle wall). Eccentric hypertrophy is associated with the increased volume load associated with regurgitant lesions.

Which of the following values may be elevated in patients with disseminated intravascular coagulation? A. Fibrinogen B. Platelet count C. Factor VIII D. D-dimer

D. D-dimer The PT, PTT, thromboplastin time, D-dimer, and fibrin degradation products are all elevated in patients with acute DIC.

Most of the decrease in hepatic blood flow associated with aging is due to A. Decreased parasympathetic activity B. Peripheral vascular disease C. Fatty infiltration of the liver D. Decreased liver mass

D. Decreased liver mass Liver mass decreases with age and accounts for most of the 20-40% decrease in hepatic blood flow with age.

Which change is principally responsible for the reduction in alveolar surface volume in the elderly? A. Osteoporosis in the spinal column B. Inflexibility of the rib cage C. COPD D. Decreased pulmonary elastic tissue

D. Decreased pulmonary elastic tissue The alveolar surface area available for gas exchange is reduced in the elderly. The cause is an overall reduction in elastic tissue in the lungs.

Which of the following is NOT one of the requirements for the diagnosis of a new myocardial infarction? A. Q waves on the ECG B. Ischemic symptoms C. Changes in serum cardiac enzyme markers D. Ejection fraction < 40%

D. Ejection fraction < 40% The diagnosis of an acute MI requires that any one of the following criteria be met: 1) the rise and fall of cardiac markers (especialy troponin), symptoms of ischemia, ECG changes indicative of ischemia, Q waves on the ECG, or imaging evidence of myocardial damage or wall motion abnormalities.

Which of the following statements regarding metabolic function in the elderly is true? A. Maximal oxygen consumption increases with age B. Circulating catecholamine levels decrease with age C. Insulin resistance decreases with age D. Elderly patients have a decreased response to beta-adrenergic agents

D. Elderly patients have a decreased response to beta-adrenergic agents Elderly patients have a decreased response to beta-adrenergic agents and an increase in circulating catecholamine levels. Maximal oxygen consumption decreases with age and insulin resistance increases, resulting in a decreased ability to handle glucose loads.

Which clotting factor mechanically stabilizes fibrin to help protect it from fibrinolysis? A. Tissue factor B. von Willebrand Factor C. Protein C D. Factor XIII

D. Factor XIII Factor XIII mechanically stabilizes fibrin to help protect it from fibrinolysis. A defect in factor XIII results in a severe bleeding disorder. Tissue factor initiates hemostasis when endothelial tissue is disrupted. Von Willebrand Factor helps bind platelets to the damaged tissue. Protein C plays a role in the anticoagulation process.

A drug with a high hepatic extraction ratio may exhibit a prolonged elimination half-life in an elderly patient because: A. Hepatic mass decreases with age B. Hepatic transaminase levels increase with age C. Cytochrome P450 activity diminishes with age D. Hepatic blood flow diminishes with age

D. Hepatic blood flow diminishes with age Hepatic transaminase levels and hepatic enzyme activity do not normally change with increased age. Hepatic mass does decrease with age, but the clearance of drugs with a high hepatic extraction ratio relies on hepatic blood flow not hepatic mass. Hepatic blood flow decreases by as much as 40% by 80 years of age.

Patients with congestive heart failure often exhibit A. Hypermagnesemia B. Hyperkalemia C. Hypernatremia D. Hyponatremia

D. Hyponatremia Patients with congestive heart failure often exhibit hyponatremia due to activation of the vasopressin system. Treatment with diuretics to reduce vascular fluid volume may also lead to hypokalemia and hypomagnesemia.

A patient experiencing a myocardial infarction also takes the drug vardenafil. What side effect could occur if he is administered nitroglycerin? A. Anaphylaxis B. Hemorrhagic stroke C. Pruritus D. Hypotension

D. Hypotension Severe hypotension may ensue if nitrates are administered to a patient taking sildenafil, tadalafil, or vardenafil.

Which of the following is not a characteristic of hypertrophic cardiomyopathy? A. Occurs frequently in patients under the age of 30 B. Is associated with diastolic dysfunction C. Patients are typically asymptomatic at rest D. Is associated with a fixed aortic obstruction

D. Is associated with a fixed aortic obstruction Patients with aortic valve stenosis have a fixed obstruction, whereas about 25% of patients with hypertrophic cardiomyopathy exhibit a dynamic obstruction which peaks in mid-to-late systole, can vary from beat to beat, and can worsen with enhanced ventricular contractility, decreased ventricular volume, and decreased left ventricular afterload. Many patients are asymptomatic, but those that do exhibit symptoms exhibit dyspnea on exertion, fatigue, syncope, or angina. The first manifestation of hypertrophic cardiomyopathy in patients younger than 30 years is often sudden cardiac death, which is also the most common cause of death. Patients with hypertrophic cardiomyopathy typically suffer from diastolic dysfunction which is exhibited by increased left ventricular end-diastolic pressures despite an often hyperdynamic left ventricular function. The diastolic stiffness is due to the increased muscle mass of the left ventricle which is typically concentrated in the upper septum just below the aortic valve.

A patient in postanesthesia recovery is exhibiting signs of an acute myocardial infarction with cardiac failure. Which of the following agents is contraindicated? A. Morphine B. Nitroglycerin C. Aspirin D. Metoprolol

D. Metoprolol Beta-blockers reduce the risk of reinfarction and death by decreasing myocardial oxygen demand. They should only be administered in a hemodynamically stable patient and should not be administered if the patient exhibits low cardiac output or heart failure.

Which calcium channel blocker can be titrated intravenously as an antihypertensive agent? A. Nifedipine B. Nimodipine C. Amlodipine D. Nicardipine

D. Nicardipine Nicardipine is a calcium channel blocker that can be titrated intravenously for use as an antihypertensive medication.

Which of the following tests is diagnostic of Alzheimer's disease? A. Positive emission tomography B. Computed tomography C. Electroencephalogram D. Postmortem analysis of the brain

D. Postmortem analysis of the brain Although computed tomography may demonstrate ventricular dilation and cortical atrophy and positive emission tomography may exhibit areas of decreased cerebral blood flow, only postmortem examination of the brain tissue is definitively diagnostic of Alzheimer's.

Which beta-blocker is available in intravenous form? A. Enalapril B. Verapamil C. Nadolol D. Propranolol

D. Propranolol In the US, propranolol, metoprolol, and esmolol are available in intravenous form. Verapamil is a calcium channel blocker. Enalapril is an ACE inhibitor.

Which of the following congenital disorders would result in obstruction of right ventricular outflow? A. Aortic stenosis B. Mitral stenosis C. Tricuspid stenosis D. Pulmonic stenosis

D. Pulmonic stenosis Pulmonic stenosis produces obstruction to right ventricular outflow.

What is the most common cause of mitral stenosis? A. Congestive heart failure B. Pulmonary hypertension C. Ebstein's anomaly D. Rheumatic heart disease

D. Rheumatic heart disease Rheumatic heart disease is the most common cause of mitral stenosis. The incidence of mitral stenosis is higher in females. Over a period of 20-30 years, rheumatic fever causes the mitral valve leaflets to become thickened, the commissure may fuse, and the leaflets and annulus may become calcified.

Which of the following responses to induction would you expect to occur in the elderly? A. Slower inhalation induction B. Increased tachycardic response to hypotension C. Increased tachycardic response to hypoxia D. Slower intravenous induction

D. Slower intravenous induction The elderly exhibit a prolonged circulation time which results in a faster inhalation induction and a slower intravenous induction time. Similar to infants, the elderly have a diminished tachycardic response to hypotension, hypovolemia, or hypoxia.

Which of the following regarding multiple sclerosis is true? A. Decreases in body temperature should be avoided B. It results from an over-proliferation of neuronal myelin C. Peripheral nerves are often affected D. The rate of relapse decreases during pregnancy

D. The rate of relapse decreases during pregnancy Multiple sclerosis is characterized by demyelination of neurons in the central nervous system (peripheral nerves are not affected). You should avoid increases in body temperature as they may result in relapses. Pregnancy is associated with a decrease in the rate of relapse.

A pacemaker that is only capable of delivering a fixed ventricular rate and does not sense spontaneous depolarizations would have a programming code of A. VVI B. VAI C. VVO D. VOO

D. VOO The letters in a pacemaker programming code in order indicate the chamber paced, chamber sensed, and whether or not the pacemaker inhibits or triggers a paced beat based on the electrical stimulus it senses. The code letters are: V = Ventricle, A = Atria, 0 = Neither, I = Inhibited, T = Triggered, and D = Dual. A pacemaker that paces the ventricle and does not sense is designated VOO.

The loss of lung elasticity seen in elderly patients results in A. a decrease in intrapleural pressure B. an increase in PaO2 C. a decrease in residual volume D. a decrease in lung recoil

D. a decrease in lung recoil The loss of lung elasticity seen in elderly patients results in a decrease in lung recoil. This, along with changes in surfactant function result in an increase in intrapleural pressure. The increased intrapleural pressure makes it more likely for small airways to collapse. The decrease in lung elastic recoil also results in an increase in the residual volume.

Which of the following does not constitute a contraindication to the use of intraoperative blood salvage (cell saver)? A. the presence of bacteria in the blood B. the presence of the topical clotting agent thrombin C. the presence of amniotic fluid in the blood D. a starting hematocrit of 45%

D. a starting hematocrit of 45% Contraindications to intraoperative blood salvage include the presence of bacteria, malignant cells, amniotic fluid, or topical clotting agents such as collagen, cellulose, and thrombin.

Postoperative cognitive dysfunction is most commonly associated with A. prior intravenous drug use B. intraoperative hypoxia C. perioperative hypotension D. advanced age

D. advanced age Progressive deterioration in overall functional connectivity of the brain and the effects of anesthetic agents in the face of those changes may explain the increased incidence of postoperative cognitive dysfunction in the elderly.

Heparin produces its anticoagulant effects by binding to A. thrombin B. thromboplastin C. factor Xa D. antithrombin

D. antithrombin Heparin is a mixture of glycosaminoglycans that produce their anticoagulant effect by binding to antithrombin (formerly known as antithrombin III). This accelerates the rate of thrombin-antithrombin binding by 1,000 to 10,000 times.

A patient develops torsades de pointes following an acute myocardial infarction. What intravenous agent is indicated for the treatment of this condition? A. potassium B. sodium bicarbonate C. calcium chloride D. magnesium

D. magnesium The effect of magnesium on mortality rates in patients with myocardial infarction is conflicted, but it is indicated in patients who develop torsades de pointes following an acute MI.

Neuroleptic malignant syndrome most closely resembles A. serotonin syndrome B. Cushing's syndrome C. narcotic overdose D. malignant hyperthermia

D. malignant hyperthermia Neuroleptic malignant syndrome is a rare, but potentially fatal complication of antipsychotic drug treatment. It is hypothesized to be caused by central nervous system depletion of dopamine. The symptoms include severe muscle rigidity, hyperpyrexia, tachycardia, dysrhythmias, hypertension, acidosis, diminished consciousness, and rhabdomyolysis. Dehydration and myoglobinuria can lead to renal failure.

What is the most serious side effect of chronic amiodarone administration? A. renal impairment B. hepatitis C. cogwheel tremor D. pulmonary toxicity

D. pulmonary toxicity The most serious side effect of chronic amiodarone administration is pulmonary toxicity resulting in alveolitis (pneumonitis). It occurs in 5-15% of patients treated with amiodarone. It is believed that amiodarone increases the production of free radicals that results in pulmonary toxicity. Because of this, it is recommended to avoid high inspired oxygen concentrations during general anesthesia for these patients as oxygen increases the production of free radicals.

Sertraline and fluoxetine are examples of A. tricyclic antidepressants B. monoamine oxidase inhibitors C. neuroleptic agents D. selective serotonin reuptake inhibitors

D. selective serotonin reuptake inhibitors Selective serotonin reuptake inhibitors (SSRIs) are the most commonly administered class of antidepressants and are the first-line drug of choice for panic disorder.

Severe heparin-induced thrombocytopenia A. typically occurs within minutes of heparin administration B. is caused by heparin antibodies to platelet factor VII C. is more common in patients with lupus erythematosus D. typically occurs after 4-5 days of heparin administration

D. typically occurs after 4-5 days of heparin administration Thrombocytopenia can develop within a few hours of heparin administration, but the severe form (resulting in a drop in the platelet count by as much as 50%) is due to heparin-dependent antibodies to platelet factor IV and typically occurs after 4-5 days of treatment.


Ensembles d'études connexes

Worksheet 14.3: Performance, Breach, and Remedies for Breach of Sales & Lease Contracts, Warranties, CISG

View Set

Chapter 5: The Complement System

View Set

Chapter 23: The Twenties (1920-1929)

View Set

Chapter 25. Depressive Disorders

View Set

CH 9 VA State Health and Insurance Exam

View Set

Common Elements Atomic Number and Atomic Mass

View Set

37 Art History: Modern Art in Latin America

View Set